Difference between revisions of "2003 AMC 12B Problems/Problem 2"

Line 4: Line 4:
 
?
 
?
 
<math> \text{(A) }-1\qquad\text{(B) }-\frac{2}{3}\qquad\text{(C) }\frac{2}{3}\qquad\text{(D) }1\qquad\text{(E) }\frac{14}{3} </math>
 
<math> \text{(A) }-1\qquad\text{(B) }-\frac{2}{3}\qquad\text{(C) }\frac{2}{3}\qquad\text{(D) }1\qquad\text{(E) }\frac{14}{3} </math>
 +
 +
==See Also==
 +
{{MAA Notice}}

Revision as of 10:24, 4 July 2013

Which of the following is the same as $(2 - 4 + 6 - 8 + 10 - 12 + 14)/ (3 - 6 + 9 - 12 + 15 - 18 + 21)$ ? $\text{(A) }-1\qquad\text{(B) }-\frac{2}{3}\qquad\text{(C) }\frac{2}{3}\qquad\text{(D) }1\qquad\text{(E) }\frac{14}{3}$

See Also

The problems on this page are copyrighted by the Mathematical Association of America's American Mathematics Competitions. AMC logo.png